שינויים

קפיצה אל: ניווט, חיפוש

חשבון אינפיניטיסימלי 2 - פתרון מועד א תשע"ג

נוספו 6,655 בתים, 13:59, 26 באוגוסט 2013
/* שאלה 6 */
==שאלה 1==
 
שאלה 1 הייתה שאלת הוכחה מההרצאה. הוכחה 4 פה: [[מדיה:Theorems2013infi2updateII.pdf|הוכחות משפטים למבחן]]
 
==שאלה 2==
נבצע הצבה <math>t=x+\frac{1}{2}</math> (רק בשביל נוחות) ואז נישאר עם
<math>\int\frac{t+\frac{3}{42}}{t^2+\frac{3}{4}}\mathrm{d}t=\frac{1}{2}\int\frac{2t+\frac{3}{2}}{t^2+\frac{3}{4}}\mathrm{d}t</math>
<math>=\frac{1}{2}\int\frac{2t}{t^2+\frac{3}{4}}\mathrm{d}t+\int\frac{\frac{3}{42}}{t^2+\frac{3}{4}}\mathrm{d}t</math>
<math>=\frac{1}{2}\ln|t^2+\frac{3}{4}|+\frac{3}{42}\frac{1}{\sqrt{\frac{3}{4}}}\arctan\frac{t}{\sqrt{\frac{3}{4}}}+c</math>
<math>=\frac{1}{2}\ln|(x+\frac{1}{2})^2+\frac{3}{4}|+\sqrt{\frac{3}{4}}\arctan\frac{x+\frac{1}{2}}{\sqrt{\frac{3}{4}}}+c</math>
ולכן <math>-\frac{2}{3}\int\frac{x+2}{x^2+x+1}\mathrm{d}x=-\frac{2}{3}(\frac{1}{2}\ln|(x+\frac{1}{2})^2+\frac{3}{4}|+\sqrt{\frac{3}{4}}\arctan\frac{x+\frac{1}{2}}{\sqrt{\frac{3}{4}}})+c</math>
אם נסכום את כל מה שקיבלנו נקבל שהתוצאה היא
<math>x+\frac{2}{3}\ln|x-1|-\frac{2}{3}(\frac{1}{2}\ln|(x+\frac{1}{2})^2+\frac{3}{4}|+\sqrt{3}\arctan\frac{x+\frac{1}{2}}{\sqrt{\frac{3}{4}}})+c</math> ואם מסדרים את זה יוצא <math>x+\frac{2}{3}\ln|x-1|-\frac{1}{3}\ln|x^2+x+1|-\frac{2}{\sqrt{3}}\arctan\frac{2x+1}{\sqrt{3}}+c</math> === סעיף ג'=== [[מדיה:Calculus2Test1Question2c2013.jpg|פתרון]] ==שאלה 3== ===סעיף א=== צריך לבדוק אם <math>\int_1^\infty \sin\sqrt{x}dx</math> מתכנס או מתבדר. הצעה לפתרון: ננסה לחשב את <math>\lim_{b\to\infty} \int_1^b \sin \sqrt{x} dx</math>. נסתכל על <math>\int \sin\sqrt x dx</math>. ע"י החלפת משתנים נקבל <math>\sqrt{x}=t \Rightarrow \frac1{2\sqrt x} dx = dt \Rightarrow dx=2tdt</math> קיבלנו <math>\int 2t\sin t dt</math>. ניתן לראות ע"י אינטגרציה בחלקים (<math>u=2t, v'=\sin t</math>) כי האינטגרל הוא <math>2\sin(t)- 2t\cos t +C</math> ולכן מתקיים: <math>\lim_{b\to\infty} \int_1^\infty \sin\sqrt x dx = \lim_{b\to\infty} (2\sin\sqrt b - 2\sqrt{b}\cos\sqrt{b})-(2\sin1-2\cos1)</math> וזה כמובן לא מתכנס ולכן האינטגרל מתבדר ===סעיף ב=== צריך לבדוק אם <math>\int_1^\infty \sin{x^{1.5}}dx</math> מתכנס או מתבדר. הצעה לפתרון: ניעזר בהצבה <math>u=x^{1.5}</math>, לכן <math>du=1.5\cdot x^{0.5} dx=1.5\sqrt[3]{u}dx</math>, כלומר <math>dx=\frac{2\cdot du}{3\cdot\sqrt[3]{u}}</math>. במקרה זה <math>u</math> בתחום <math>[1,\infty]</math> גם כן. לכן: <math>\int_1^\infty \sin{x^{1.5}}dx=\frac{2}{3}\int_1^\infty \frac{\sin{u}}{\sqrt[3]{u}}du</math>, והאינטגרל שהתקבל מתכנס על פי דיריכלה. ==שאלה 4== '''הפרכה:''' ניקח את <math>f_n(x)=\left\{\begin{matrix} \frac1n,\ \ x=0\\ 0 \ \ \ \ \ \ \mathrm{else} \end{matrix}\right.</math> וההתכנסות היא במ"ש (קל להוכיח). עוד פונקציה שמפריכה היא <math>f_n(x)=\frac{D(x)}{n}</math> כאשר <math>D(x)</math> היא פונקציית דיריכלה. זאת אומרת, <math>D(x)=\left\{\begin{matrix} 1,\ \ x\in \mathbb{Q}\\ 0 \ \ \ \ \ \ \mathrm{else} \end{matrix}\right.</math> ==שאלה 5== ===סעיף א'=== קודם כל, נראה כי <math>\sum_{n=1}^\infty \frac{(-1)^n(x^n-x^{n+1})}{2^n+n^3}=\sum_{n=1}^\infty \frac{(-1)^nx^n(1-x)}{2^n+n^3}=(1-x)\sum_{n=1}^\infty \frac{(-1)^nx^n}{2^n+n^3}</math> וקיבלנו טור חזקות כיוון ש- <math>(1-x)</math> קבוע וכפל בקבוע לא משנה התכנסות או התבדרות של טור. כעת נסתכל על החלק של טור החזקות בלבד. ניתן לחשב את רדיוס ההתכנסות שלו לפי דלאמבר: <math>R=\lim_{n\to\infty} |\frac{a_n}{a_{n+1}}|=\lim_{n\to\infty} |\frac{\frac{(-1)^n}{2^n+n^3}}{\frac{-1\cdot(-1)^n}{2\cdot2^n+(n+1)^3}}|=\lim_{n\to\infty}\frac{2\cdot 2^n +(n+1)^3}{2^n+n^3}=2</math> כעת, נשאר לנו רק לבדוק התכנסות ב- <math>x=2,-2</math>. ניתן לראות בקלות שהאיבר הכללי של הסדרה לא ישאף ל-0 ב-2 המקרים האלה ולכן תנאי הכרחי להתכנסות של טור לא מתקיים. ניתן להסיק שהטור מתכנס אם ורק אם <math>x\in(-2,2)</math> ===סעיף ב'=== דבר ראשון, נפרק את הפונקציה באופן הבא: <math>f(x)=\frac{1+x}{(1-x^2)^2}=\frac{1}{(1-x^2)^2}+x\cdot\frac{1}{(1-x^2)^2}</math>. כעת נזכור כי <math>\frac{1}{(1-x)^2}=(\frac{1}{1-x})'=(\sum_{n=0}^\infty x^n)'=\sum_{n=0}^\infty nx^{n-1}</math> והפעולה של החלפת הנגזרת והסכום חוקית כיוון שהטור ההנדסי מתכנס במ"ש. ולכן, <math>\frac{1}{(1-x^2)^2}=\sum_{n=1}^\infty n(x^2)^{n-1}=\sum_{n=1}^\infty n\cdot x^{2n-2}</math> ו- <math>x\cdot\frac{1}{(1-x^2)^2}=x\cdot\sum_{n=1}^\infty n(x^2)^{n-1}=\sum_{n=1}^\infty n\cdot x^{2n-1}</math> לכן ניתן להגיע לכך ש- <math>f(x)=\sum_{n=1}^\infty (nx^{2n-1}+nx^{2n-2})</math>. כעת כל מה שנישאר זה קצת לשחק עם המקדמים והמשתנים ככה שיצא טור מקלורן. (יש לשים לב שהמקדמים של הטור יהיו 1,1,2,2,3,3... ולכן נשתמש ב- ceil או floor לתאר את המקדמים)  *הצעה אחרת לפתרון ל-5ב [[מדיה:Question5BCalculus2Test1Year2013.pdf|כאן]]  * אפשר לפתור את התרגיל הזה גם בגישה ישירה: <math>\frac{1+x}{(1-x^2)^2}=\frac{1}{1-x}\frac{1}{1-x^2}</math> ידוע ש: <math>\frac{1}{1-x}=1+x+x^2+\ldots</math> <math>\frac{1}{1-x^2}=1+x^2+x^4+\ldots</math> ולכן  <math>\frac{1}{1-x}\frac{1}{1-x^2}=(1+x+x^2+\ldots)(1+cx^2+x^4+\ldots)</math> <math>=1(1+x+x^2+\ldots)+x^2(1+x+x^2+\ldots)+x^4(1+x+x^2+\ldots)+\ldots=(1+x+x^2+\ldots)+(x^2+x^3+x^4+\ldots)+(x^4+x^5+x^6+\ldots)+\ldots</math> ומכאן קל לראות שהמקדמים צריכים להיות <math>1,1,2,2,3,3</math> וכו' ==שאלה 6== נזכור כי הנוסחה לחישוב אורך עקומה של <math>f(x)</math> בקטע <math>[a,b]</math> היא <math>\int_a^b \sqrt{1+(f'(x))^2} dx</math> ולכן אנחנו מחפשים את <math>\int_0^{\frac{\pi}6}\sqrt{1+((\ln\cos(x))')^2}dx</math>. מתקיים: <math>\frac{d}{dx}\ln\cos(x) = \frac{-\sin(x)}{cos(x)}=-\tan(x)</math>. כמו כן, <math>1+(-\tan(x))^2=1+\frac{\sin^2(x)}{\cos^2(x)}=\frac1{\cos^2(x)}</math>. נראה כי <math>\forall x \in [0,\frac{\pi}6] :\cos(x)>0</math> ולכן <math>\sqrt{\frac{1}{\cos^2(x)}}=\frac1{\cos(x)}</math> ולא צריך לדאוג לגבי הסימן של המכנה (שורש תמיד חיובי ויש מקרים בהם דווקא <math>-\cos(x)</math> חיובי בעוד ש- <math>\cos(x)</math> שלילי) נרצה לחשב כעת את <math>\int_0^{\frac{\pi}6} \frac{1}{\cos(x)} dx</math> נעזר בהצבה אוניברסאלית <math>t=tan(\frac { x }{ 2 } )</math>, ולכן האינטגרל הוא <math>\int _{ 0 }^{ \frac { \pi }{ 6 } }{ \frac { 1 }{ cosx } dx } =\int _{ 0 }^{ tan(\frac { \pi }{ 12 } ) }{ \frac { 1 }{ \frac { 1-t^{ 2 } }{ t^{ 2 }+1 } } \cdot \frac { 2dt }{ t^{ 2 }+1 } } =\int _{ 0 }^{ tan(\frac { \pi }{ 12 } ) }{ \frac { 2 }{ 1-t^{ 2 } } dt } =\int _{ 0 }^{ tan(\frac { \pi }{ 12 } ) }{ \frac { 2 }{ 1-t^{ 2 } } dt } =\int _{ 0 }^{ tan(\frac { \pi }{ 12 } ) }{ [\frac { 1 }{ t+1 } -\frac { 1 }{ t-1 } ]dt } =</math><math>=[ln|t+1|-ln|t-1|\quad ]_{ 0 }^{ tan(\frac { \pi }{ 12 } ) }=ln|\frac { t+1 }{ t-1 } |\quad ]_{ 0 }^{ tan(\frac { \pi }{ 12 } ) }=0.5493...</math>
307
עריכות